Câu hỏi:

41 lượt xem

Chọn câu trả lời đúng nhất để điền vào chỗ chấm: (MN)2=...{\left( {M - N} \right)^2} = ...

(NM)2{\left( {N - M} \right)^2}.
M22MN+N2{M^2} - 2MN + {N^2}.
N22NM+M2{N^2} - 2NM + {M^2}.
Cả ABC đều đúng.

Xem đáp án

Lời giải

Hướng dẫn giải:

Đáp án đúng là: D

Ta có \({\left( {M - N} \right)^2} = {M^2} - 2MN + {N^2}\) (bình phương của một hiệu) nên đáp án B đúng.

Vì \({\left( { - 1} \right)^2} = 1\) nên \({\left( {M - N} \right)^2} = {\left( { - 1} \right)^2}.{\left( {M - N} \right)^2} = {\left[ { - \left( {M - N} \right)} \right]^2} = {\left( {N - M} \right)^2}\) nên đáp án A đúng.

Lại có \({\left( {N - M} \right)^2} = {N^2} - 2NM + {M^2}\), do đó đáp án C đúng.

Vậy cả 3 đáp án ABC đều đúng.

CÂU HỎI HOT CÙNG CHỦ ĐỀ